Warum sind bestimmte Drehungen instabil? (Euler-Gleichungen)

Wir haben die Euler-Gleichungen für einen rotierenden Körper wie folgt

ich 1 ω ˙ 1 + ω 2 ω 3 ( ich 3 ich 2 ) = 0 ich 2 ω ˙ 2 + ω 1 ω 3 ( ich 1 ich 3 ) = 0 ich 3 ω ˙ 3 + ω 2 ω 1 ( ich 2 ich 1 ) = 0
Woher ich ich sind die Trägheitsmomente über die x ich Achse und ω ich ist die Winkelgeschwindigkeit um diese Achse.

Es kann gezeigt werden (*), dass wenn ich 1 > ich 2 > ich 3 , dann Objekte mit Winkelgeschwindigkeit sehr nahe ω = ( 0 , 1 , 0 ) sind instabil. Warum ist das so und wie kann ich versuchen, es mir vorzustellen?

Ich habe versucht, mir dies anhand eines Balls vorzustellen, aber festgestellt, dass dies wahrscheinlich keine gute Möglichkeit ist, es zu visualisieren, da ein Ball kugelsymmetrisch ist, sodass die Trägheitsmomente nicht unterschiedlich sind. Gibt es eine Visualisierung oder Animation, die es mir ermöglichen könnte, diese Drehung zu sehen und möglicherweise zu verstehen, warum sie instabil ist?


(*) Als Antwort auf den Kommentar von @SRS:

Ich bin mir bei Referenzen nicht sicher, aber ich weiß, wie es geht: Let ω 1 = η 1 , ω 3 = η 3 wo η ist eine kleine Störung, und nehme an ω 2 = 1 + η 2 . Dann werden die Euler-Gleichungen

(1) ich 1 η ˙ 1 = ( ich 2 ich 3 ) η 3 + Ö ( η 2 )
(2) ich 2 η ˙ 2 = Ö ( η 2 )
(3) ich 3 η ˙ 3 = ( ich 1 ich 2 ) η 1 + Ö ( η 2 )
Unterscheiden ( 1 ) und Sub ein ( 3 ) zum resultierenden Ausdruck
η ¨ 1 = ( ich 2 ich 3 ) ( ich 1 ich 2 ) ich 3 ich 1 η 1
Wenn ich 1 > ich 2 > ich 3 , dann ist die Konstante auf der rechten Seite positiv, also ist die Lösung dieser Gleichung eine Exponentialfunktion (wenn es eine andere Ordnung wäre, dann wäre die Lösung a Sünde / cos ). Daher ist es instabil.


Bearbeiten:

Zur Verdeutlichung habe ich diese Frage gepostet, um andere visuellere Möglichkeiten zum Verständnis dieses Effekts zu sehen, anstatt die Gleichungen wie oben zu lösen, und um zu sehen, wie dieser Effekt im wirklichen Leben ins Spiel kommt. Ich glaube also nicht, dass es sich um ein Duplikat der anderen Fragen handelt, da sie keine passenden Antworten haben.

Du hast es gerade gezeigt haha. WENN Sie es testen möchten, ist der beste Weg, wie Sie sagen, ein Objekt mit auszuwählen ich 1 ich 2 ich 3 ich 1 , und eine gute 1 ist eine Taschentuchbox oder irgendein Prisma.
@FGSUZ Ja, ich weiß, wie man es mathematisch zeigt, meine Frage ist, wie man es visualisiert, wie zum Beispiel die Links von ZeroTheHero
Sie können dies auch selbst mit einem Schuh demonstrieren.
Mögliche Duplikate: Physics.stackexchange.com/q/67957/2451 , Physics.StackExchange.com /q/17504/2451 , Physics.StackExchange.com /q/34364/2451 und Links darin.
@Qmechanic - Ich stimme zu, dass dies ein Duplikat dieser älteren Fragen ist, aber ich mag die Antworten hier besser als auf alle anderen. Vielleicht sollten diese älteren Fragen als Duplikat davon geschlossen werden.
@Qmechanic Ich hatte eine dieser Fragen bereits bei der Suche gesehen, aber die anderen nicht gefunden. Unabhängig davon glaube ich nicht, dass es sich um ein Duplikat handelt, da ich ausdrücklich nach einer visuelleren Methode gefragt habe, um dies zu verstehen, anstatt nur zu zeigen, dass eine kleine Störung zu einer exponentiellen Lösung führt.
Eine weitere gute Visualisierung besteht darin, einen langen Stab um eine Achse zu drehen, die nahe an seiner Mittelachse liegt, aber nicht dieselbe wie sie ist. Wenn Sie sich genau um diese Mittelachse drehen, ist sie stabil. Aber wenn Sie ein wenig daneben liegen, ziehen die Rotationskräfte die Stange weiter von dieser Achse weg.

Antworten (3)

Es gibt eine andere schöne Möglichkeit, dies mathematisch zu sehen. Es ist nicht allzu schwer zu zeigen, dass es im Körperrahmen zwei Erhaltungsgrößen gibt: das Quadrat des Drehimpulsvektors

L 2 = L 1 2 + L 2 2 + L 3 2
und die kinetische Rotationsenergie, die sich herausstellt
T = 1 2 ( L 1 2 ich 1 + L 2 2 ich 2 + L 3 2 ich 3 ) .
(Beachten Sie, dass der Drehimpuls L selbst wird nicht im Körperrahmen konserviert; aber sein Quadrat ist zufällig eine Konstante.)

Wir können dann die Frage stellen: Für gegebene Werte von L 2 und T , was sind die zulässigen Werte von L ? Das ist leicht zu sehen L 2 Einschränkung bedeutet das L muss auf der Oberfläche einer Kugel liegen; und es ist fast so einfach zu sehen, dass die T Einschränkung bedeutet das L muss auch auf der Oberfläche eines gegebenen Ellipsoids mit Hauptachsen liegen 2 T ich 1 > 2 T ich 2 > 2 T ich 3 . Somit sind die erlaubten Werte von L muss auf dem Schnittpunkt einer Kugel und eines Ellipsoids liegen. Wenn wir halten L 2 fest und erzeugen Sie eine Reihe dieser Kurven für verschiedene Werte von T , sie sehen so aus:

Geben Sie hier die Bildbeschreibung ein

Beachten Sie, dass für einen gegebenen Wert von L 2 , hat ein Körper seine höchstmögliche kinetische Energie, wenn er sich um die Achse mit dem niedrigsten Trägheitsmoment dreht , und umgekehrt.

Nehmen wir also an, dass sich ein Körper um die Achse seines höchsten Trägheitsmoments dreht. Wenn wir dieses Objekt so stören, dass wir seine Energie leicht ändern (um der Argumentation willen annehmen, dass L 2 konstant bleibt), sehen wir, dass der Vektor L wird jetzt auf einer relativ kleinen Kurve in der Nähe seiner ursprünglichen Position liegen. Wenn sich das Objekt um seine Achse der niedrigsten Trägheit dreht, L bleibt relativ nahe an seinem ursprünglichen Wert, wenn es gestört wird.

Deutlich anders sieht es jedoch aus, wenn sich das Objekt zunächst um die Zwischenachse dreht (dritter roter Punkt im Diagramm oben, auf der „Vorderseite“ der Kugel. Die Konturen sind leicht gestört T in der Nähe dieses Punktes bleiben Sie nicht in der Nähe der Zwischenachse; Sie wandern über die ganze Sphäre. Es gibt also nichts zu halten L davon abzuhalten, über diese Sphäre zu wandern, wenn wir das Objekt etwas von der Drehung um diese Achse weg stören; was impliziert, dass ein Objekt, das sich um seine Zwischenachse dreht, instabil ist.

So hatte ich das noch nicht gesehen. Ich finde das Diagramm immer noch ziemlich abstrakt, aber nachdem ich die Videos über das T-Bar-Zeug gesehen habe, wird es etwas klarer - die Kurven folgen den größeren ellipsenartigen Linien für die instabilen Schwingungen. Vielen Dank! :)
Schöne Antwort.
Diese Antworten – Ihre und die von ZeroTheHero – sind einfach wunderschön. Ich habe noch nie in diesen Begriffen darüber nachgedacht: Ich war nur froh zu wissen, dass eine stabile Linearisierung eines Systems um einen Gleichgewichtspunkt herum bedeutet, dass man eine Kugel mit einem Radius ungleich Null finden kann, in der man das Problem als kontraktives Fixum umwandeln kann Punkt Problem. Aber - solche Schönheit auch aus globaler Sicht!
Hey Michael, ich habe herumgespielt und versucht, ein Programm zu schreiben, um eine Handlung wie diese zu produzieren. Ich habe es geschafft, eine zu bekommen, die wie die Handlung von ZeroTheHero aussieht, aber ich kann nicht herausfinden, wie ich Ihre Figur zeichnen soll. Haben Sie die allgemeine Form der Gleichung der gelben Kurven berechnet und diese dann gezeichnet? Wenn ja, erinnerst du dich, welche das waren?
@JohnDoe: Für diese Version der Handlung habe ich die Form der Kurven tatsächlich explizit ausgearbeitet; Sie passen jedoch nicht in dieses Kommentarfeld. :-) Effektiv drückt man sich aber aus L 1 , L 2 , L 3 in Kugelkoordinaten, schreibe beide Flächen in Kugelkoordinaten auf, setze sie einander gleich, verwende die Identität Sünde 2 θ = ( 1 cos 2 θ ) , und dann auflösen nach cos 2 θ als Funktion von ϕ . Wenn Sie jedoch Mathematica verwenden, ist es einfacher, die MeshFunctionsOption mit der kinetischen Energie als Ihre zu verwenden MeshFunction.

Es gibt eine Alternative zur @MichaelSeifert-Methode, die Drehimpuls und Trägheitsmomente verwendet: Sie muss sich mit dem Vektor befassen ω direkt, da wir an der Evolution dieses Vektors interessiert sind.

Man kann die kinetische Energie und die Länge zum Quadrat ausdrücken L als

T = 1 2 ( ich 1 ω 1 2 + ich 2 ω 2 2 + ich 3 ω 3 2 ) L L = ich 1 2 ω 1 2 + ich 2 2 ω 2 2 + ich 3 2 ω 3 2
Diese beiden Gleichungen definieren unterschiedliche Ellipsoide in ω -Platz. Da es sich um drei Komponenten handelt ω aber nur zwei Einschränkungen, die Einschränkungen sind nicht genug, um vollständig zu bestimmen ω . Vielmehr die Entwicklung von ω muss auf dem Schnittpunkt dieser beiden Ellipsen liegen, wie durch die schwarze Linie in der Abbildung dargestellt. (Die grüne Ellipse ist konstant L L und die gelbe Konstante T .)

Geben Sie hier die Bildbeschreibung ein

Dieser Schnittpunkt hier hat ungefähr die Form einer abgeflachten Banane, wobei die lange Seite der Banane in der ω 2 Richtung.

So wurde während der Evolution die Spitze von ω kann sich entlang dieser Kreuzung bewegen und gleichzeitig die Energie behalten und L L Konstante. Die Figur veranschaulicht den Fall, wo

ich 1 = 4 , ich 2 = 3 / 2 , ich 3 = 1
mit T = 5 / 2 und | L | = 2.45 .

Sie können aus der Abbildung ersehen, dass, wenn wir an der Kreuzung oben beginnen, die Entwicklung von ω 3 ist im Bereich ziemlich eingeschränkt (und kann das Vorzeichen nicht ändern), dass die Entwicklung von ω 1 ist auch ziemlich eingeschränkt, aber dass die Komponente ω 2 ist viel größer (im Grunde die gesamte Länge der abgeflachten Banane). Daher ist qualitativ gesehen die Drehung um die Mittelachse instabil , weil ω 2 muss nicht in der Nähe seines Anfangswerts bleiben.

(Dasselbe Argument gilt für die Komponenten L k = ω k ich k in Michael Seiferts Figur, nur dass in seinem Fall L 1 würde das Vorzeichen aber nicht ändern L 3 könnte in der Nähe des hyperbolischen Fixpunkts das Vorzeichen ändern.)

Und das ist auch eine schöne Antwort. Übrigens, sobald Sie dies gelernt haben und es in einem Buch um seine Mittelachse herum ausprobieren und es herumtaumeln sehen, werden Sie es nie vergessen. Das beste Buch dafür ist Goldsteins Classical Mechanics
Diese Antworten – deine und Michaels – sind einfach wunderschön. Ich habe noch nie in diesen Begriffen darüber nachgedacht: Ich war nur froh zu wissen, dass eine stabile Linearisierung eines Systems um einen Gleichgewichtspunkt herum bedeutet, dass man eine Kugel mit einem Radius ungleich Null finden kann, in der man das Problem als kontraktives Fixum umwandeln kann Punkt Problem. Aber - solche Schönheit auch aus globaler Sicht!
Dies ist in der Tat die traditionellere Methode, und in diesem Fall werden die Schnittkurven Polhoden genannt (eines meiner Lieblingsworte in der Physik). Ich finde es nur etwas einfacher zu visualisieren, wenn eine der Begrenzungsflächen eine Kugel ist .
@MichaelSeifert Ich habe heute noch etwas gelernt. Vielen Dank.

Mein Physikprofessor half uns, das mit einem Tennisschläger zu visualisieren.

Das Drehen mit der Achse entlang des Griffs ist stabil.

Das Spinnen in der Ebene des Schlägers ist stabil. John McEnroe hat seine Schläger oft so umgedreht.

Sich in die andere Richtung zu drehen, wie Sie einen Schläger schwingen würden, ist instabil. Egal wie vorsichtig Sie den Schläger umdrehen, er dreht sich auch in die andere Richtung, bevor er wieder auf Ihre Hand kommt.

Falls Sie keinen Schläger zum Spielen haben: https://www.youtube.com/watch?v=4dqCQqI-Gis

Vielen Dank! Bis ich diese Frage gestellt habe, war mir nicht klar, dass dies manchmal als Tennisschläger-Theorem bezeichnet wird. Wenn man das gewusst hätte, wäre es viel einfacher gewesen, es im wirklichen Leben zu visualisieren